Show that the stationary paths of this system satisfy the Euler-Lagrange equation d²y β dx² +\w(x)y=0, y(0) = 0, (a− yλ) y(1) + ßy′(1) = 0, where X is a Lagrange multiplier. Consider the functional S[y] = ay(1)² + | " dx ẞy², y(0) = 0, with a natural boundary condition at x = 1 and subject to the constraint 1 C[y] = ry(1)² + [[" dx w(x) y² = 1, where a, ẞ and y are nonzero constants.

Calculus For The Life Sciences
2nd Edition
ISBN:9780321964038
Author:GREENWELL, Raymond N., RITCHEY, Nathan P., Lial, Margaret L.
Publisher:GREENWELL, Raymond N., RITCHEY, Nathan P., Lial, Margaret L.
Chapter9: Multivariable Calculus
Section9.3: Maxima And Minima
Problem 20E
Question
Show that the stationary paths of this system satisfy the
Euler-Lagrange equation
d²y
β
dx²
+\w(x)y=0, y(0) = 0, (a− yλ) y(1) + ßy′(1) = 0,
where X is a Lagrange multiplier.
Transcribed Image Text:Show that the stationary paths of this system satisfy the Euler-Lagrange equation d²y β dx² +\w(x)y=0, y(0) = 0, (a− yλ) y(1) + ßy′(1) = 0, where X is a Lagrange multiplier.
Consider the functional
S[y] = ay(1)² + | "
dx ẞy², y(0) = 0,
with a natural boundary condition at x = 1 and subject to the constraint
1
C[y] = ry(1)² + [["
dx w(x) y² = 1,
where a, ẞ and y are nonzero constants.
Transcribed Image Text:Consider the functional S[y] = ay(1)² + | " dx ẞy², y(0) = 0, with a natural boundary condition at x = 1 and subject to the constraint 1 C[y] = ry(1)² + [[" dx w(x) y² = 1, where a, ẞ and y are nonzero constants.
Expert Solution
steps

Step by step

Solved in 2 steps with 1 images

Blurred answer